Difference between revisions of "2014 AMC 12B Problems/Problem 20"

(Created page with "The domain of the LHS implies that <cmath>40<x<60</cmath> Begin from the left hand side <cmath>log_{10}[(x-40)(60-x)]<2</cmath> <cmath>-x^2+100x-2500<0</cmath> <cmath>(x-50)^2>0<...")
 
Line 1: Line 1:
 
The domain of the LHS implies that <cmath>40<x<60</cmath> Begin from the left hand side
 
The domain of the LHS implies that <cmath>40<x<60</cmath> Begin from the left hand side
<cmath>log_{10}[(x-40)(60-x)]<2</cmath>
+
<cmath>\log_{10}[(x-40)(60-x)]<2</cmath>
 
<cmath>-x^2+100x-2500<0</cmath>
 
<cmath>-x^2+100x-2500<0</cmath>
 
<cmath>(x-50)^2>0</cmath>
 
<cmath>(x-50)^2>0</cmath>
 
<cmath>x \not = 50</cmath>
 
<cmath>x \not = 50</cmath>
 
Hence, we have integers from 41 to 49 and 51 to 59. There are 18 integers.
 
Hence, we have integers from 41 to 49 and 51 to 59. There are 18 integers.

Revision as of 17:48, 20 February 2014

The domain of the LHS implies that \[40<x<60\] Begin from the left hand side \[\log_{10}[(x-40)(60-x)]<2\] \[-x^2+100x-2500<0\] \[(x-50)^2>0\] \[x \not = 50\] Hence, we have integers from 41 to 49 and 51 to 59. There are 18 integers.